Question

Please show all work AND any calculator functions. The table below summarizes data of heights and...

Please show all work AND any calculator functions.

The table below summarizes data of heights and weights of 8 randomly selected adults. Use this information to answer parts a) – h).

Height (x) in inches 5.2 5.6 5.8 5.9 5.4 6.1 6.0 5.7

Weight (y) in pounds 119 136 155 185 135 202 194 165

a) Test the claim ρ ≠ 0. Use α = 0.01.

• State the hypotheses and label the claim.

• State the significance level (α).

• Is the test left-tailed, right-tailed, or two-tailed (circle one)?

• What is the critical value(s)?

• What are the P-value and test-statistic?

• State the conclusion

b) Find the linear regression equation.

c) Graph the regression equation on a scatter plot.

d) Find the correlation coefficient. Make a conclusion about the strength of the correlation.

e) Give the coefficient of determination.

f) What percentage of the variation in y is explained by the variation in x?

g) What percentage of the variation in y is unexplained by the variation in x?

h) Predict y when x = 5.85

Given Answers to match= a) H0: ρ = 0 H1: ρ ≠ 0 (claim) α = 0.01; two-tailed; critical values: r = 0.834, t = ±3.707; test statistics: P-value = 0.0002, r = 0.9539, t = 7.786 Reject the null hypothesis. The P-value is less than α, the test statistic t is greater than 3.707 and falls in the critical region, and the test statistic r is greater than 0.834 and falls in the critical region. There is sufficient evidence to support the claim that ρ ≠ 0 and there is a linear correlation between height and weight. b) y = -382.4682 + 95.2023 d) r = 0.9539 strong positive correlation f) r2 = 0.9099 g) 90.99% explained h) 100% - 90.99% = 9.01% is unexplained i) y = 174.465

0 0
Add a comment Improve this question Transcribed image text
Answer #1
X Y XY
5.2 119 618.8 27.04 14161
5.6 136 761.6 31.36 18496
5.8 155 899 33.64 24025
5.9 185 1091.5 34.81 34225
5.4 135 729 29.16 18225
6.1 202 1232.2 37.21 40804
6.0 194 1164 36 37636
5.7 165 940.5 32.49 27225
Ʃx = 45.7
Ʃy = 1291
Ʃxy = 7436.6
Ʃx² = 261.71
Ʃy² = 214797
Sample size, n = 8
x̅ = Ʃx/n = 45.7/8 = 5.7125
y̅ = Ʃy/n = 1291/8 = 161.375
SSxx = Ʃx² - (Ʃx)²/n = 261.71 - (45.7)²/8 = 0.64875
SSyy = Ʃy² - (Ʃy)²/n = 214797 - (1291)²/8 = 6461.875
SSxy = Ʃxy - (Ʃx)(Ʃy)/n = 7436.6 - (45.7)(1291)/8 = 61.7625

a) Null and alternative hypothesis:

Ho: ρ = 0 ; Ha: ρ ≠ 0

α = 0.01

Correlation coefficient, r = SSxy/√(SSxx*SSyy) = 61.7625/√(0.64875*6461.875) = 0.9539

It is a two tailed test.

df = n-2 = 6

Critical value, t_c = T.INV.2T(0.01, 6) = 3.707

Test statistic :  

t = r*√(n-2)/√(1-r²) = 0.9539 *√(8 - 2)/√(1 - 0.9539²) = 7.786

p-value = T.DIST.2T(ABS(7.7861), 6) = 0.0002

Conclusion:

p-value < α Reject the null hypothesis. There is a correlation between x and y.

b)

Slope, b = SSxy/SSxx = 61.7625/0.64875 = 95.202312

y-intercept, a = y̅ -b* x̅ = 161.375 - (95.20231)*5.7125 = -382.4682

Regression equation :

ŷ = -382.4682 + (95.2023) x

c) Graph:

d) Correlation coefficient, r = SSxy/√(SSxx*SSyy) = 61.7625/√(0.64875*6461.875) = 0.9539

It has a strong positive correlation

e) Coefficient of determination, r² = (SSxy)²/(SSxx*SSyy) = (61.7625)²/(0.64875*6461.875) = 0.9099

f) Explained = 90.99%

g) Unexplained = 9.01%

h) Predicted value of y at x = 5.85

ŷ = -382.4682 + (95.2023) * 5.85 = 174.4653

Add a comment
Know the answer?
Add Answer to:
Please show all work AND any calculator functions. The table below summarizes data of heights and...
Your Answer:

Post as a guest

Your Name:

What's your source?

Earn Coins

Coins can be redeemed for fabulous gifts.

Not the answer you're looking for? Ask your own homework help question. Our experts will answer your question WITHIN MINUTES for Free.
Similar Homework Help Questions
  • Linear Regression and Prediction perform a linear regression to determine the line-of-best fit. Use weight as...

    Linear Regression and Prediction perform a linear regression to determine the line-of-best fit. Use weight as your x (independent) variable and braking distance as your y (response) variable. Use four (4) places after the decimal in your answer. Sample size, n:     21 Degrees of freedom: 19 Correlation Results: Correlation coeff, r: 0.3513217 Critical r:           ±0.4328579 P-value (two-tailed): 0.11837 Regression Results: Y= b0 + b1x: Y Intercept, b0:      125.308 Slope, b1:            0.0031873 Total Variation:       458.9524 Explained Variation:   56.6471 Unexplained Variation: 402.3053...

  • Question 5: reject or do not reject Question 6: 0.10, 0.05, and 0.01; 0.10, 0.05, 0.01,...

    Question 5: reject or do not reject Question 6: 0.10, 0.05, and 0.01; 0.10, 0.05, 0.01, and 0.001; 0.10 only; none no, some, strong, very strong, extremely strong Using the appropriate model, sample size n, and output below: Model: y = 8 + 81X1 + 82X2 + 83x3 + € Sample size: n = 16 S = .7727 Sq = 88.6% Sq(adj) = 84.0% Analysis of Variance Source DF Regression 2 Residual Error 5 Total 7 F P 19.48 0.0044...

  • What would you predict the highway mpg for a car that is 142 inches long? Show...

    What would you predict the highway mpg for a car that is 142 inches long? Show your calculation and round to the nearest whole number. Analysis Data Data Sets Window Help St ClearCopy PasteSave relation and Regression Print Significance: 0.05 Select the columns to be used for the x and y variables Sample size, n: Degrees of freedom: 19 21 Correlation Results: Correlation coeff, r: -0.7327824 Criticalr: 9 20.43285 79 x variable columny variable column P-value (two-tailed): 0.00016 Regression Results:...

  • Please Show Work. Thank you. 2. Conduct a hypothesis test showing all of your steps. A...

    Please Show Work. Thank you. 2. Conduct a hypothesis test showing all of your steps. A Gallup Poll reports that 65% of American adults support labor unions. In order to test this claim, 988 adults in the U.S. were surveyed, of which 617 approved of labor unions. Do 65% of Americans support them? Test this claim with a 6% level of significance. a) State the hypotheses and label the claim. b) Is it left-tailed, right-tailed, or two-tailed (circle one)? c)...

  • Bighorn sheep are beautiful wild animals found throughout the western United States. Let x be the...

    Bighorn sheep are beautiful wild animals found throughout the western United States. Let x be the age of a bighorn sheep (in years), and let y be the mortality rate (percent that die) for this age group. For example, x = 1, y = 14 means that 14% of the bighorn sheep between 1 and 2 years old died. A random sample of Arizona bighorn sheep gave the following information: x 1 2 3 4 5 y 15.8 19.3 14.4...

  • The data below shows the selling price in hundred thousands) and the list price in hundred...

    The data below shows the selling price in hundred thousands) and the list price in hundred thousands) of homes sold. A StatDisk output yields the following screen. Degrees of freedom Korrelation Results Korrelation coeff, F: 0.9916979 Critical ri 0.6118972 P-value (two-tailed): 0.000 Regression Results Jy bo bi Y Intercept, b01 0.9156039 slope, bit 1.025779 Total Variations 34791.6 Explained variation 34216.31 Unexplained Variation: 575.2891 Standard Error! 8.480043 Coeff of Bet, RA2: 0.9834647 What is the regression equation? y = 0.916 +...

  • Please Show Work. Thank you. 7. Conduct a hypothesis test showing all of your steps. Females...

    Please Show Work. Thank you. 7. Conduct a hypothesis test showing all of your steps. Females under the age of 51 years old are supposed to get, on average, 18 mg of iron daily. In a random sample of 44 women, their daily mean intake was 16.77 mg of iron with a standard deviation of 3.02 mg. An agency claims that women get less than the recommended daily iron intake. Test this claim with a 4% level of significance. a)...

  • (a) Suppose n = 6 and the sample correlation coefficient is r=0.894. IS significant at the...

    (a) Suppose n = 6 and the sample correlation coefficient is r=0.894. IS significant at the 1% level of significance (based on a two-tailed test)? (Round your answers to three decimal places.) critical Conclusion: Yes, the correlation coefficient p is significantly different from 0 at the 0.01 level of significance. No, the correlation coefficient p is not significantly different from 0 at the 0.01 level of significance. (b) Suppose n = 10 and the sample correlation coefficient is r =...

  • ***PLEASE DO ALL THIS IS MY LAST QUESTION OF THE GIVEN AMOUNT OF QUESTIONS :( *** Which value of r indicates a stronger correlation:r 0.751 orr0.896? Explain your reasoning Choose the correct answer...

    ***PLEASE DO ALL THIS IS MY LAST QUESTION OF THE GIVEN AMOUNT OF QUESTIONS :( *** Which value of r indicates a stronger correlation:r 0.751 orr0.896? Explain your reasoning Choose the correct answer below. A. r=-0.896 represents a stronger correlation because 0.751 >-0.896. B. r=0.751 represents a stronger correlation because l-08961-10.751. C· r--0 896 represents a stronger correlation because 1-0 8961-10751. D. 0.751 represents a stronger correlation because 0.751> -0.896. The budget (in millions of dollars) and worldwide gross (in...

ADVERTISEMENT
Free Homework Help App
Download From Google Play
Scan Your Homework
to Get Instant Free Answers
Need Online Homework Help?
Ask a Question
Get Answers For Free
Most questions answered within 3 hours.
ADVERTISEMENT
ADVERTISEMENT
ADVERTISEMENT